Calculate the volume of space between the two cylinders in the diagram . Use 3.14 for pi! Show work !

Calculate The Volume Of Space Between The Two Cylinders In The Diagram . Use 3.14 For Pi! Show Work !

Answers

Answer 1

Answer:

116933.6

Step-by-step explanation:

[tex]A_1 = \pi r^2 = 3.14 * 34^2 = 3.14 * 1156 = 3629.84[/tex]

[tex]V_1 = A_1 * h = 3629.84 * 40 = 145193.6[/tex]

[tex]A_2 = \pi r^2 = 3.14 * 15^2 = 3.14 * 225 = 706.5[/tex]

[tex]V_2 = A_2 *h = 706.5 * 40 = 28260[/tex]

[tex]V = V_1 - V_2 = 145193.6 - 28260 = 116933.6[/tex]


Related Questions

Which of these statements is correct? The system of linear equations 6 x minus 5 y = 8 and 12 x minus 10 y = 16 has no solution. The system of linear equations 7 x + 2 y = 6 and 14 x + 4 y = 16 has an infinite number of solutions. The system of linear equations 8 x minus 3 y = 10 and 16 x minus 6 y = 22 has no solution. The system of linear equations 9 x + 6 y = 14 and 18 x + 12 y = 26 has an infinite number of solutions

Answers

Answer:

The only true statement is:

"The system of linear equations 8x - 3y = 10 and 16x - 6y = 22 has no solution."

Step-by-step explanation:

First, some definitions.

A system of linear equations has infinite solutions if both equations define the same line, has no solutions if we have two parallel lines, has one solution in all the other cases.

Where two lines are parallel if we can write them as:

a*x + b*y = c

a*x + b*y = d

where c and d are different numbers.

Now we can analyze the given statements:

a)

6x - 5y = 8

12x - 10y = 16

has no solution?

If we divide both sides of the second equation by 2, we get:

(12x - 10y)/2 = 16/2

6x - 5y = 8

We get the first equation, then both equations define the same line, thus the system has infinite solutions, then the statement is false.

b)

7x + 2y = 6

14x + 4y = 16

has infinite solutions?

Let's divide the second equation by 2, then we get:

(14x + 4y)/2 = 16/2

7x + 2y = 8

If we rewrite our system of equations, we get:

7x + 2y = 6

7x + 2y = 8

These are parallel lines, thus, this system has no solutions.

So the statement is false.

c)

8x - 3y = 10

16x - 6y = 22

has no solution?

Again, let's divide the second equation by 2 to get:

(16x - 6y)/2 = 22/2

8x - 3y = 11

If we rewrite our system:

8x - 3y = 10

8x - 3y = 11

These are parallel lines, thus the system has no solutions, so this statement is correct.

d)

9x + 6y = 14

18x + 12y = 26

Has infinite solutions?

Dividing the second equation by 2 we get:

(18x + 12y)/2 = 26/2

9x + 6y = 13

So the equations are different (are parallel lines again) so this system has not infinite solutions.

Then the statement is false.

Answer:

The answer to your question is the third choice.

Step-by-step explanation:

a)          6x - 5y = 8

          12x - 10y = 16

We observe that these lines are the same so they have infinite solutions.

b)

           7x + 2y = 6

          14x + 4y = 16

These lines are parallel because they have the same slope, so they do not cross, there is no solution.

c)

            8x - 3y = 10

           16x - 6y = 22

These lines are parallel because they have the same slope, so they do not cross, there is no solution.

d)

           9x + 6y = 14

          18x + 12y = 26

These lines are parallel because they have the same slope, so they do not cross, they do not have an infinite number of solutions.

Find all real zeros of the function y = -7x + 8

Answers

9514 1404 393

Answer:

  x = 8/7

Step-by-step explanation:

The only real zero of this linear function is the value of x that makes y=0:

  0 = -7x +8

  7x = 8 . . . . . . add 7x

  x = 8/7 . . . . . .divide by 7

A golfer hits a golf ball.
The function
d(t) = –2t2 + 7t + 4
most closely represents the height(h) of the golf ball in feet after t seconds. How
long is the golf ball in the air?

Answers

Answer:

The golf ball was in the air for 4 seconds.

Step-by-step explanation:

Solving a quadratic equation:

Given a second order polynomial expressed by the following equation:

[tex]ax^{2} + bx + c, a\neq0[/tex].

This polynomial has roots [tex]x_{1}, x_{2}[/tex] such that [tex]ax^{2} + bx + c = a(x - x_{1})*(x - x_{2})[/tex], given by the following formulas:

[tex]x_{1} = \frac{-b + \sqrt{\Delta}}{2*a}[/tex]

[tex]x_{2} = \frac{-b - \sqrt{\Delta}}{2*a}[/tex]

[tex]\Delta = b^{2} - 4ac[/tex]

In this question:

We have to find the amount of time it takes for the ball to hit the ground. We have that:

[tex]d(t) = -2t^2 + 7t + 4[/tex]

Which is a quadratic equation with [tex]a = -2, b = 7, c = 4[/tex].

How long is the golf ball in the air?

We have to find t for which [tex]d(t) = 0[/tex]

So

[tex]-2t^2 + 7t + 4 = 0[/tex]

[tex]\Delta = b^{2} - 4ac = (7)^2 - 4(-2)(4) = 81[/tex]

[tex]t_{1} = \frac{-7 + \sqrt{81}}{2*(-2)} = -0.5[/tex]

[tex]t_{2} = \frac{-7 - \sqrt{81}}{2*(-2)} = 4[/tex]

Time is a positive measure, so t = 4.

The golf ball was in the air for 4 seconds.

A triangle has three vertices at the points A = (0, -2), B = (3, 4), and C = (-1.5, 2.5). Prove that this triangle is a right-angle triangle, using algebra. (Note: a right angle triangle has one 90 degree angle).

Answers

Answer:eraf

Step-by-step explanation:

What is the zero of the function represented by this graph?

Answers

answer: -5
explanation: the x-intercept is (-5,0), the x value of the x intercept is the zero.

Please help show the steps
Please put 15 years old

Answers

Answer:

P = $98.77

Step-by-step explanation:

FV = p (1+i)^n -1

            i

pv = 700,000

i = .075/12 = .00625

n = (66 - 15)* 12 = 612

700,000 = P (( 1 + .00625)^ 612 -1 /.00625

4375 = P (1.00625)^612 -1)

P = $98.77

Answer:

page 1:

51 years

$98.78

639546.64 (i think)

Page 2:

213 months

17.8 years

321 months

26.8 years

1128.9 months

88.8 years

I would probably choose the second plan because it's rather unlikely that i live past 90

Step-by-step explanation:

page 1

Let's assume the payments are at the end of the month

66-15= 51 years

effective rate: .075/12=.00625

[tex]700000=x\frac{(1+.00625)^{51*12}-1}{.00625}\\x=98.77973387[/tex]

which i guess we can round to 98.78

700000-98.78*(51*12)= 639546.64

This number is really really high and so maybe you want to double check it

page 2

effective rate: .051/12=.00425

[tex]700000=5000\frac{1-(1+.00425)^{-n}}{.00425}\\.405=(1+.00425)^{-n}\\log_{1.00425}.405=-n\\n=213[/tex]

213 months

213/12= 17.8 years

[tex]700000=4000\frac{1-(1+.00425)^{-n}}{.00425}\\.25625=(1.00425)^{-n}\\log_{1.00425}.25625\\n=321[/tex]

321 months

321/12=26.8 years

[tex]700000=3000\frac{1-(1+.00425)^{-n}}{.00425}\\.008333333=(1.0045)^{-n}\\log_{1.0045}.00833333=-n\\n=1128.9[/tex]

1128.9 months

1128.9/12= 94.1 years

1066 months

1066/12= 88.8 years

Longhorn Pizza has the following number of topping options available: four vegetables, two meats, and two cheeses. A pizza is ordered with exactly four toppings. What is the probability that the pizza is ordered with exactly two vegetables, one meat, and one cheese

Answers

Answer:

The probability is [tex]\frac{24}{70}[/tex].

Step-by-step explanation:

topping options available: four vegetables, two meats, and two cheeses

Number of topping on one pizza = 4

Getting two vegetables = (4 C 2)

Getting one meat = (2 C 1)

Getting one cheese = (2 C 1)

Choosing 4 toppings out of 8 = (8 C 4)

probability that the pizza is ordered with exactly two vegetables, one meat, and one cheese

[tex]\frac{(4C2)\times (2C1)\times (2C1)}{(8C4)}\\\\\frac{6\times 2\times 2}{70}\\\\\frac{24}{70}[/tex]

Patel squeezed oranges so that his family could have fresh-squeezed juice for breakfast. He squeezed StartFraction 4 over 17 EndFraction cups from the first orange, StartFraction 3 over 10 EndFraction cups from the second orange, StartFraction 9 over 20 EndFraction cups from the third orange, StartFraction 3 over 11 EndFraction cups from the fourth orange, and StartFraction 7 over 15 EndFraction cups from the fifth orange. Patel estimates that he needs 3 cups of orange juice for his family. About how much more orange juice does he need to reach his estimate?

Answers

Answer:

A. 1/2 cups

Step-by-step explanation:

13/15 is close to 1

1/5 is a small amount

9/20 is just over 1/2

5/11 is just under 1/2

7/15 is just under 1/2

Estimate: 1 + 1/2 + 1/2 + 1/2 + a little = 2 1/2

He needs 3 cups, so he needs another 1/2 cup.

Answer: A. 1/2 cups

Answer:

it a 1/2

Step-by-step explanation:

Find the area of this circle. Use 3 for T.
Α = πη2
5 in
[?] in?

Answers

Hope this help!!!

Have a nice day!!!

SOMEONE HELP ME PLEASE
find the real fifth root of -32

Answers

Answer:  -2

This is because (-2)^5 = -32. Applying the fifth root to both sides lets us say [tex]-2 = \sqrt[5]{-32}[/tex]

There are four other roots but they are complex. Effectively, we are solving the equation [tex]x^5 + 32 = 0[/tex]

PLEASE HELP!!!

WILL MARK BRAINLIEST!!!

If the diameter of the circle shown below is 6ft and 0 is a right angle, what is the length of segment AB to the nearest foot?

Multiple choice!

Thank you!

Answers

Answer:

how old are you gghhjjzetstu9u

Answer:

4 ft

Step-by-step explanation:

let's find radius first

radius=diameter/2

=6/2

=3 ft

radii=3 ft

Now by using pythagoras theorem

a^2 + b^2 = c^2

3^2 + 3^2 =AB^2

9+9=AB^2

18=AB^2

[tex]\sqrt{18}[/tex] AB

4.24 =AB

4 ft =AB (after converting to nearest foot)

The number of adults who attend a music festival, measured in hundreds of people, is represented by the function a(d)=−0.3d2+3d+10, where d is the number of days since the festival opened.

The number of teenagers who attend the same music festival, measured in hundreds of people, is represented by the function t(d)=−0.2d2+4d+12, where d is the number of days since the festival opened.

What function, f(d) , can be used to determine how many more teenagers than adults attend the festival on any day?


f(d)=−0.1d2+d+22

f(d)=0.1d2+d+2

f(d)=−0.1d2+7d+2

f(d)=0.1d2+7d+2

Answers

Answer:

f(d)=0.1d^2+d+2

Step-by-step explanation:

t(d)=−0.2d2+4d+12

a(d)=−0.3d2+3d+10

how many more teenagers than adults attend the festival on any day?

==>

f(d) = t(d) - a(d)

=0.1d^2+d+2

PLEASE HELP ME I HAVE TO PASS THIS TEST

30 POINTS

Answers

Answer:

Hi, there the answer is

These are the equations with exactly one solution

-5x + 12 = –12x – 12

-5x + 12 = 5x + 12

-5x + 12 = 5x – 5

Hope This Helps :)

Step-by-step explanation:

First degree equations

A first degree equation has the form

ax + b = 0

There are some special cases where the equation can have one, infinitely many or no solution

If , the equation has exactly one solution

If a=0 and b=0 the equation has infinitely many solutions, because it doesn't matter the value of x, it will always be true that 0=0

If a=0 and  the equation has no solution, because it will be equivalent to b=0 and we are saying it's not true. No matter what x is, it's a false statement.

We have been given some equations, we only need to put them in standard form

-5x + 12 = –12x – 12

Rearranging

7x + 24 = 0

It has exactly one solution because a is not zero

.......................

-5x + 12 = 5x + 12

Rearranging

-10x + 0 = 0

It has exactly one solution because a is not zero

.......................

-5x + 12 = 5x – 5

Rearranging

-10x + 17 = 0

It has exactly one solution because a is not zero

.......................

-5x + 12 = -5x – 12

Rearranging

0x + 24 = 0

It has no solution, no matter what the value of x is, it's impossible that 24=0

Answer: These are the equations with exactly one solution

-5x + 12 = –12x – 12

-5x + 12 = 5x + 12

-5x + 12 = 5x – 5

Hi the answer is -5x+12=-12x-12

The endpoints of DEF are D(1, 4) and F(16, 14).
Determine and state the coordinates of point E, if
DE: EF = 2:3.

Answers

Answer:

The coordinates of point E are (7,8).

Step-by-step explanation:

Point E:

Is given by (x,y).

DE: EF = 2:3.

This means that, for both coordinates x and y:

[tex]E - D = \frac{2}{2+3}(F-D)[/tex]

[tex]E - D = \frac{2}{5}(F-D)[/tex]

x-coordinate:

x-coordinate of D: 1

x-coordinate of F: 16

[tex]E - D = \frac{2}{5}(F-D)[/tex]

[tex]x - 1 = \frac{2}{5}(16-1)[/tex]

[tex]x - 1 = 2*3[/tex]

[tex]x = 7[/tex]

y-coordiante:

y-coordinate of D: 4

y-coordinate of F: 14

[tex]E - D = \frac{2}{5}(F-D)[/tex]

[tex]y - 4 = \frac{2}{5}(14-4)[/tex]

[tex]y - 4 = 2*2[/tex]

[tex]x = 8[/tex]

The coordinates of point E are (7,8).

WHAT is the answer? Pls Time limit

Answers

Answer:

11 is the answer.....

Step-by-step explanation:

6x-15+39+90 = 180

What is the mean of the data?

Answers

Answer:

The mean (average) of a data set is found by adding all numbers in the data set and then dividing by the number of values in the set. The median is the middle value when a data set is ordered from least to greatest. The mode is the number that occurs most often in a data set.

the ratio of the length of a rectangular floor to its width is 3:2 if the length of the floor is 12 meters what is the perimeter of the floor in meters

Answers

9514 1404 393

Answer:

  40 meters

Step-by-step explanation:

The ratio of width to length is 2/3, so the width of the floor is ...

  width = (2/3)(12 m) = 8 m

The perimeter is found from ...

  P = 2(L +W)

  P = 2(12 m +8 m) = 40 m

The perimeter of the floor is 40 m.

_____

Additional comment

As above, the perimeter is twice the sum of length and width. In terms of ratio units, that is p = 2(3 +2) = 10. The length is 3 ratio units, so the perimeter is 10/3 times the length. (10/3)(12 m) = 40 m.

I need the answer. Please help me

Answers

C = 25 deg

a ≈ 10.72 ft

b ≈ 11.83 ft

Answer:

C = 25

a = 10.72

b = 11.83

Step-by-step explanation:

C:

Solve; 180 - (90 + 65)

a:

Tan(65) = a/5

Tan(65) * 5 = a

10.72 = a

b:

Cos(65) = 5/b

Cos(65) * b = 5

b = 5 / Cos(65)

b = 11.83

Hope this helps!

Which statements are true of the function f(x) = 3(2.5)x? Check all that apply.

The function is exponential.
The initial value of the function is 2.5.
The function increases by a factor of 2.5 for each unit increase in x.
The domain of the function is all real numbers.
The range of the function is all real numbers greater than 3.

Answers

Answer:

A) The function is exponential.

C) The function increases by a factor of 2.5 for each unit increase in x.

D) The domain of the function is all real numbers.

Step-by-step explanation:

Got it right on Edge :)

Answer:

a,c,d are correct

Step-by-step explanation:

Lori downloaded all the pictures she took at Rita’s wedding into a single computer folder. She took 86 of the 134 pictures with her camera and the remainder of them with her cell phone. Of the pictures Lori took with her cell phone, one out of every five was blurry.

Answers

Answer:

87

Step-by-step explanation:

PLEASE WILL MARK IF YOU HELP!!

Answers

Answer:

22°

63°

m<H=22°

m<G=63°

Problem 1

Answer:   79

--------------------------

Work Shown:

For any triangle, the three angles always add to 180

For any isosceles triangle, the base angles are congruent. The base angles are opposite the congruent sides. We see that angle O = angle H.

O+H+T = 180

H+H+T = 180

2H+T = 180

2H+22 = 180

2H = 180 - 22

2H = 158

H = 158/2

H = 79

=======================================================

Problem 2

Answer:   54

--------------------------

Work Shown:

We'll use the same ideas as problem 1.

In this case, angle O = angle D = 63 since they are the base angles opposite the congruent sides.

D+G+O = 180

63+G+63 = 180

G+126 = 180

G = 180-126

G = 54

You deposit $10,000 in an account earning 4% interest compounded monthly. a. How much will you have in the account in 25 years? b. How much interest will you earn?

Answers

Answer:

In 25 years I will have $ 27,137.65. Therefore, I will earn $17,137.65 interest.

Step-by-step explanation:

Given that I deposit $ 10,000 in an account earning 4% interest compounded monthly, to determine how much will you have in the account in 25 years and how much interest will I earn, the following calculation must be performed:

10,000 x (1 + 0.04 / 12) ^ 25x12 = X

10,000 x (1 + 0.00333) ^ 300 = X

10,000 x 2.7137 = X

27,137.65 = X

Therefore, in 25 years I will have $ 27,137.65. Therefore, I will earn $17,137.65 interest.

Find the surface area of each solid figure

Answers

Answer:

First find the SA of the triangular figure

4 x 3 = 12 cm^2 (the triangles on the sides)

2 x 3 = 6 cm^2 (the back square)

2 x 5 = 10 cm^2 (the slanted square)

*I'm not sure if this question includes the bottom of the triangle but here it is anyways

4 x 2 = 8 cm^2

Including the bottom the SA of the triangular figure is:

12 + 6 + 10 + 8 = 36 cm^2

Find the SA of the rectangular shape

4 x 2 = 8 cm^2 (the bottom square)

2 x 6 = 12 x 2 = 24 cm^2 (the sides)

4 x 6 = 24 x 2 = 48 cm^2 (the front and back)

Add them up

8 + 24 + 48 = 80 cm^2

If you wanted to find the SA of the whole figure it would be:

12 + 6 + 10 + 8 + 24 + 48 = 108 cm^2

Hope this helps!

Jai bought a helmet and a pair of skates.
The helmet cost £45.
He sold both items for £224.
Jai made a 120% profit on the cost of the helmet and a 40% profit on the total cost.
What was the percentage profit on the skates?
Give your answer to 1 decimal place.

Answers

Answer:

Profit % on skates = 8.7 %

Step-by-step explanation:

Step 1 : Find cost price of skates

Cost price of helmet = £45

Let cost price of skate be = x

Selling price = £224

Cost price = (x + 45)

Total profit % = 40%

[tex]Profit \% = \frac{Selling \ price - cost \ price }{Cost \ price} \times 100[/tex]

[tex]\frac{40}{100} = \frac{224 - (x + 45)}{(x + 45)}\\\\40(x+ 45) = 100(224 - (x +45))\\\\40(x + 45) = 22400 - 100(x + 45)\\\\40(x +45) + 100(x+ 45) = 22400\\\\140(x + 25) = 22400\\\\x + 45 = \frac{22400}{140}\\\\x = 160 - 45 = \£ \ 115[/tex]

Total cost price = 45 + 115 = £160

Step 2 : Selling price of Helmet

Cost price of Helmet = £45

Let selling price of helmet be = y

Profit % of helmet = 120 %

[tex]Profit \% = \frac{selling \ price - cost \ price}{cost \ price}[/tex]

[tex]\frac{120}{100} = \frac{y -45}{45}\\\\\frac{120 \times 45}{100} = y -45\\\\54 = y - 45\\\\99 = y[/tex]

Step 3 : Selling price of skates

Total selling = selling price of helmet + selling price of skates

224 = 99 + selling price of skates

224 - 99 = selling price of skates

125 = selling price of skates

Step 4 : Profit percentage on skates

Cost price of skate = £ 115

Selling price of skate = £ 125

[tex]Profit \% \ on \ skates = \frac{selling\ price- cost \ price }{cost \ price} \times 100[/tex]

                           [tex]= \frac{125-115}{115} \times 100\\\\=\frac{10}{115} \times 100\\\\= 8.7 \%[/tex]

Use implicit differentiation to find an equation of the tangent line to the curve at the given point. y2(y2 − 4) = x2(x2 − 5) (0, −2) (devil's curve)

Answers

Answer:

Step-by-step explanation:

Given that:

[tex]y^2 (y^2-4) = x^2(x^2 -5)[/tex]

at point (0, -2)

[tex]\implies y^4 -4y^2 = x^4 -5x^2[/tex]

Taking the differential from the equation above with respect to x;

[tex]4y^3 \dfrac{dy}{dx}-8y \dfrac{dy}{dx}= 4x^3 -10x[/tex]

Collect like terms

[tex](4y^3 -8y)\dfrac{dy}{dx}= 4x^3 -10x[/tex]

[tex]\dfrac{dy}{dx}= \dfrac{4x^3 -10x}{4y^3-8y}[/tex]

Hence, the slope of the tangent line m can be said to be:

[tex]\dfrac{dy}{dx}= \dfrac{4x^3 -10x}{4y^3-8y}[/tex]

At point (0,-2)

[tex]\dfrac{dy}{dx}= \dfrac{4(0)^3 -10(0)}{4(-2)^3-8-(2)}[/tex]

[tex]\dfrac{dy}{dx}= \dfrac{0 -0}{4(-8)+16}[/tex]

[tex]\dfrac{dy}{dx}= 0[/tex]

m = 0

So, we now have the equation of the tangent line with slope m = 0 moving through the point (x, y) = (0, -2) to be:

(y - y₁ = m(x - x₁))

y + 2 = 0(x - 0)

y + 2 = 0

y = -2

3. Find the value of X (In the picture) (giving points to best answer/brainlest)​

Answers

Answer:

101 =x

Step-by-step explanation:

The measure of the exterior angle is equal to the sum of the opposite interior angles

143 = 42+x

Subtract 42 from each side

143-42 = 42+x-42

101 =x

Answer:

x = 101 degrees

Step-by-step explanation:

The sum of the external angle and its adjacent is 180 degrees

143 + y = 180

y = 37 degrees

The sum of the inner angles of a triangle is 180 degrees

37 + 42 + x = 180

79 + x = 180

x = 101 degrees

m.ng giúp mình về phần vector trong ma trận nha

Answers

Answer:

maybe if u translate it in English

Step-by-step explanation:

it wouldv been helpful if u mind?

A
2x+5
x² + 5x + 6
x² + 5x+6
B
2x+5​

Answers

Answer:

what is the question?

Step-by-step explanation:

answer the question

To start a shop Rajeev and sneha invested $2,75,280 and $5,35,870 .if they needed $ 8,85,356 .How much money should they need more ?​

Answers

Answer:

$74,206

Step-by-step explanation:

Total money : $2,75,280 + $5,35,870 = $8,11,150

Amount needed : $8,85,356 - $8,11,150 = $74,206

Use the slope formula to find the slope of the line through the points (2,10) and (10,−8).

Answers

The slope formula is the changes of two y-values over/to the changes of two x-values.

[tex] \large \boxed{m = \frac{y_2 - y_1}{x_2 - x_1} }[/tex]

Substitute two given points in the formula to find the slope. The m-term represents the slope from y = mx+b.

[tex]\large{m = \frac{10 - ( - 8)}{2 - 10} } \\ \large{m = \frac{10 + 8}{ - 8} } \\ \large{ m = \frac{18} { - 8} \longrightarrow \frac{9}{ - 4} } \\ \large \boxed{m = - \frac{9}{4} }[/tex]

Answer

The slope is -9/4.

Hope this helps and let me know if you have any doubts!

Answer:

m=-9/4

Step-by-step explanation:

Hi there!

The formula for the slope (m) calculated from two points is given as (y2-y1)/(x2-x1), where (x1,y1) and (x2,y2) are points

we are given the two points (2,10) and (10,-8)

to avoid any confusion, let's label the values of the points

x1=2

y1=10

x2=10

y2=-8

now substitute into the formula:

m=(-8-10)/(10-2)

subtract

m=(-18)/(8)

simplify (reduce to lowest terms)

m=-9/4

Hope this helps!

Other Questions
Help find value of X please! The operating income calculated using variable costing and absorption costing amounts to $9,100 and $11,200. There were no beginning inventories. Determine the total fixed manufacturing overhead that will be expensed under absorption costing for the year. Upon decomposition, a sample of sodium bicarbonate produces 0.0118 g of sodium carbonate, Na2CO3. How many grams of water does it produce Which of the following statements is false? Question 21 options: Not all economists are agreed as to whether government should bail out companies in financial trouble. Not all economists prefer a rule-based monetary policy to discretionary monetary policy. Rule-based monetary policy advocates often assert that discretionary monetary policy can be motivated by politics. The tax multiplier is always larger than the government spending mult I need the answer pls help its in scince Phoebe has 4 bags of cat food. She feeds her cat 1/5 of a bageach day. Find the number of days that will pass before Phoeberuns out of cat food. Draw a model to show how you foundyour answer. ANSWER PLSS The following are all negative factors of urban sprawl EXCEPTO the wasting of land, including prime agricultural tracts.O the subdivision of houses for low-income families.O the possibility of high infrastructure costs to get power to isolated areas.O the use of more energy, as motor vehicles are required for most trips.O the decline of inner cities because of people choosing to move out. 1.Implied meaning is the idea an author suggests without saying it outright.TrueFalse2.Short stories have long character descriptions.TrueFalse3.We make Christian judgments when we decide what is right and wrong according to God's Word.TrueFalse4.An author writes to entertain, to inform, to give facts, directions, or influence thinking. We call the author's reason for writing the author's _____.authoritypurposeskill5.An important reading comprehension skill is to identify the setting, characters, and _____.life styleplotdescriptions Lines DE and AB intersect at point C. What is the value of x? 6(x + 2) in the simplest form How do you turn off notifications for a particular question that you're following?I dont want to turn off notifications for the whole app, just the ones for a certain discussion because they wont stop talking. Which of the following was a result of Joseph McCarthys attack on President Truman in the early 1950s?(Summer school) Why is blood considered a suspension? which pair of sentences best completes the diagram showing the course of the French Revoluyion? . Match the following countries with their capital city.1. Colombia2. Ecuado3. Bolivia4. Venezuela5. Pera. Bogotb. La Pazc. Caracasd. Limae. Quito 2. Which of the following equations represents a neutralization reaction?a. H2CO3 + H2O + CO2b. 2H2O2 2H2O + O2c. Ba(NO3)2 + K2CO3 BaCO3 + 2KNO3d. HNO3 + NaOH NaNO3 + H2O Hey, could you please help me? thank you What did Adam encourage Daniel to do? the table shows the varying prices of microphones sold at makes music machine prices of microphones (dollar's) 25,28,20,22,32,28,35,34,30,36part bthe table shows varying prices at macks music machine. find the mean absolute deviation of the data show your work What is the best description of the message of this political cartoon? Giving women the right to vote will cause problems and humiliate men. Giving women the right to vote will help relieve men of the burden of power. When women gain the right to vote, the country will benefit in many ways. Women gaining the right to vote will have little impact, positive or negative.